The Flat Earth Society

Flat Earth Discussion Boards => Flat Earth Debate => Topic started by: Yashas on April 04, 2017, 08:35:42 AM

Title: [Ask Me Anything] Will explain every observation using round earth model
Post by: Yashas on April 04, 2017, 08:35:42 AM
This is an open challenge. Provide me with any evidence that disproves round earth and I'll give an explanation using mathematics and physics on why it is wrong.

The only assumption I am going to make is that the mathematics and physics I have been taught is true.

Note: Observations must be related to FE/RE models. Arbitrary nonsense won't be appreciated. For example,

Why do men have nipples?
Title: Re: Evidence to prove Round Earth is wrong - I'll prove that the evidence is wrong
Post by: Jonny B Smart on April 04, 2017, 08:42:03 AM
You will like my inverse tangent thread.
Title: Re: Evidence to prove Round Earth is wrong - I'll prove that the evidence is wrong
Post by: FalseProphet on April 04, 2017, 08:49:59 AM
Hmm...can you explain me why I see different star constellations at different latitudes while the angular distances between the stars do not change?

Also, why are there 2 celestial poles. And don't tell me there aren't, I live near the equator.
Title: Re: Evidence to prove Round Earth is wrong - I'll prove that the evidence is wrong
Post by: Yashas on April 04, 2017, 09:01:57 AM
If you are in the northern hemisphere, you cannot see the stars which are below your horizon. You cannot see the north star from the southern hemisphere.

(http://en.es-static.us/upl/2012/03/ecliptic_640.jpg)

The angular distance changes but you cannot measure it without accurate equipment. The stars are very very far away! Trillions of miles away! The stars are moving and the constellations won't appear the same in a few thousand years. You can google for predicted pictures of famous constellations after thousands of years.

Celestial poles refer to the celestial sphere. This is an imaginary sphere around the earth. As the celestial sphere is a sphere, it has two poles.

Astronomy Introduction Crash Course: https://www.youtube.com/playlist?list=PLX2gX-ftPVXWddG6sGq6H-3cN0IQzfQmG
Astronomy Understanding Night Sky Crash Course: https://www.youtube.com/playlist?list=PLX2gX-ftPVXU3BfWLJfmXfGD08cIit5CI

Understanding celestial sphere:

You will be fully convinced why you see different stars at different latitudes after you go through the crash course.

I am not sure if you are a FEer. If you thought I was fighting for FE, then you were confused. I fixed the very confusing title :P
Title: Re: Evidence to prove Round Earth is wrong - I'll prove that the evidence is wrong
Post by: FalseProphet on April 04, 2017, 09:08:40 AM
Sorry, the title of the thread is a bit misleading and I maybe did not read the OP. I thought you want to disprove RE using physics and math and I really wanted to see how you do that.

Can you? Just for me?
Title: Re: Evidence to prove Round Earth is wrong - I'll prove that the evidence is wrong
Post by: Novarus on April 04, 2017, 02:01:00 PM
Sorry, the title of the thread is a bit misleading and I maybe did not read the OP. I thought you want to disprove RE using physics and math and I really wanted to see how you do that.

Can you? Just for me?

Other way around - Yashas is a spherical Earth proponent - they're saying any flat earth conjecture can be explained and improved upon to match observations to a spherical model.

In short, give me a Flat Earth scenario and you will get an explanation of how it doesn't work and how the spherical Earth does.
Title: Re: [Ask Me Anything] Will explain every observation using round earth model
Post by: disputeone on April 04, 2017, 02:33:13 PM
Re: [Ask Me Anything] Will explain every observation using round earth model

Rotational speed of outer section of galaxies?
Title: Re: [Ask Me Anything] Will explain every observation using round earth model
Post by: physical observer on April 04, 2017, 02:34:43 PM
This is an open challenge. Provide me with any evidence that disproves round earth and I'll give an explanation using mathematics and physics on why it is wrong.

The only assumption I am going to make is that the mathematics and physics I have been taught is true.

Why is there a waterfall on Lake Victoria, Africa, where the said centrifugal forces are bulging the water out to make the earth globe? If the earth's rotation is bulging the water out at the equator, how can it also fall to a lower spot on earth? If the water at the equator is being bulged out to make the globe, why is the large Lake Victoria, sitting at the equator, so level and horizontal across its surface?

http://vizts.com/wp-content/uploads/2016/04/Victoria-Falls-amazing-view.jpg
Title: Re: [Ask Me Anything] Will explain every observation using round earth model
Post by: Novarus on April 04, 2017, 03:46:32 PM
This is an open challenge. Provide me with any evidence that disproves round earth and I'll give an explanation using mathematics and physics on why it is wrong.

The only assumption I am going to make is that the mathematics and physics I have been taught is true.

Why is there a waterfall on Lake Victoria, Africa, where the said centrifugal forces are bulging the water out to make the earth globe? If the earth's rotation is bulging the water out at the equator, how can it also fall to a lower spot on earth? If the water at the equator is being bulged out to make the globe, why is the large Lake Victoria, sitting at the equator, so level and horizontal across its surface?

http://vizts.com/wp-content/uploads/2016/04/Victoria-Falls-amazing-view.jpg

Lake Victoria is subject to gravity AND centrifugal force.
We keep coming back to this - why do you think physical forces are mutually exclusive?

And while were at it, have you been there?

Now let's use your reasoning:
How do we know that's a picture of Lake Victoria? We have to take your word for it. There is no way for us to verify that that is what you say it is.
Just another empty assertion with fake pictures.

If you want to use photographic evidence to support your claims, you can't discount any of the photos that have been used to discount yours.
Title: Re: [Ask Me Anything] Will explain every observation using round earth model
Post by: Son of Orospu on April 04, 2017, 03:47:04 PM
Why do men have nipples?
Title: Re: [Ask Me Anything] Will explain every observation using round earth model
Post by: JackBlack on April 04, 2017, 03:48:08 PM
Why is there a waterfall on Lake Victoria, Africa, where the said centrifugal forces are bulging the water out to make the earth globe? If the earth's rotation is bulging the water out at the equator, how can it also fall to a lower spot on earth? If the water at the equator is being bulged out to make the globe, why is the large Lake Victoria, sitting at the equator, so level and horizontal across its surface?
The apparent acceleration due to the apparent centrifugal force is roughly 0.03 m/s^2 (I can do the math again if you want, even showing the derivation of it).
The acceleration due to gravity is roughly -9.8 m/s^2.
This meas the net acceleration is roughly -9.8 m/s^2 and thus the water will fall.

The lake is not horizontal across its surface. If you wish to claim it is you will need to provide evidence it is, evidence which is capable of showing how flat or curved it is and to what degree of uncertainty.

It is level with the apparent gravitational potential, which is a combination of the gravitational potential and the apparent centrifugal force.
Title: Re: [Ask Me Anything] Will explain every observation using round earth model
Post by: JackBlack on April 04, 2017, 04:03:34 PM
Why do men have nipples?
Because they are mutant women.
Title: Re: [Ask Me Anything] Will explain every observation using round earth model
Post by: Semnomic on April 04, 2017, 06:02:49 PM
This is an open challenge. Provide me with any evidence that disproves round earth and I'll give an explanation using mathematics and physics on why it is wrong.

The only assumption I am going to make is that the mathematics and physics I have been taught is true.

OK I know you can answer most problems as you are smart but please tell me this

which come first the chicken or the egg on ball earth  :-X lol
Title: Re: [Ask Me Anything] Will explain every observation using round earth model
Post by: Semnomic on April 04, 2017, 06:11:08 PM
This is an open challenge. Provide me with any evidence that disproves round earth and I'll give an explanation using mathematics and physics on why it is wrong.

The only assumption I am going to make is that the mathematics and physics I have been taught is true.

Serious one - At high alt, right on boundary of space.

What stops small particles escaping? they must be colliding, moving about so whats stopping them just moving that little bit more over that line and floating off?

Could that be happening but they are getting replaced by inbound particles kind of a constant balancing act ?

Mars got me thinking about this.
Title: Re: [Ask Me Anything] Will explain every observation using round earth model
Post by: Novarus on April 04, 2017, 06:26:27 PM
This is an open challenge. Provide me with any evidence that disproves round earth and I'll give an explanation using mathematics and physics on why it is wrong.

The only assumption I am going to make is that the mathematics and physics I have been taught is true.

OK I know you can answer most problems as you are smart but please tell me this

which come first the chicken or the egg on ball earth  :-X lol

Egg-laying animals evolved before the chicken - the first chicken as we know it hatched from an egg - the egg came first.
Can we get back to the debate?
Title: Re: [Ask Me Anything] Will explain every observation using round earth model
Post by: Novarus on April 04, 2017, 06:49:24 PM
This is an open challenge. Provide me with any evidence that disproves round earth and I'll give an explanation using mathematics and physics on why it is wrong.

The only assumption I am going to make is that the mathematics and physics I have been taught is true.

Serious one - At high alt, right on boundary of space.

What stops small particles escaping? they must be colliding, moving about so whats stopping them just moving that little bit more over that line and floating off?

Could that be happening but they are getting replaced by inbound particles kind of a constant balancing act ?

Mars got me thinking about this.

It's a combination of various things: Gravity, the Earth's magnetic field and the cycles of the dynamic atmosphere.
Watch the flat earth proponents telling me that any source I post is part of the great conspiracy.
However, for those serious about debate:
https://en.wikipedia.org/wiki/Solar_wind#Atmospheres
http://sciencing.com/earths-magnetosphere-protects-suns-solar-wind-1955.html - this one actually talks about Mars too
https://en.wikipedia.org/wiki/Atmospheric_escape#Dominant_atmospheric_escape_and_loss_processes_on_Earth

Yes, I'm using wikipedia - it's laid out concisely and simply and if you want more information there are countless links to follow. But since I ask more of people coming at this from the other side, I'm also going to explain how I understand what's going on:

The Earth's gravity keeps the atmosphere close to it, just like it does with the moon and the plethora of space junk accruing in orbit - much of which can be observed with nothing more than a pair of binoculars if you know where to look. On top of that, the dynamic centre of the earth produces a magnetic field that keeps us safe from the ravages of the near-vacuum of space. Hot metal spinning fast produces electromagnetic currents - this is a scientific fact you can verify yourself if you really want to (but this isn't a discussion of electrodynamics - go to a physics board if you want to argue that.) Plus, anyone who has used a compass has observed the effect of the Earth's magnetic field.
The most commonly observable consequences of this solar wind hitting the magnetosphere are the aurorae which happen above both the north and the south pole. This does send charged particles into the atmosphere, but this isn't where we get the matter back. Most of the air lost to the solar wind is replenished by the Earth itself. On top of that, oxygen gas is too massive to be stripped away by the solar wind quickly thanks to a combination of the magnetosphere and Earth's gravity. Over astronomical time periods, the atmosphere is being stripped away but due to the size of the Earth, this effect is negligible on human time scales.

Incidentally, in the Flat Earth model, those aurorae should be visible from everywhere on Earth. This is not the case.

tl;dr - the particles do escape, but it's happening really slowly because the Earth is really big and has a magnetic field

Title: Re: [Ask Me Anything] Will explain every observation using round earth model
Post by: Semnomic on April 04, 2017, 07:21:17 PM
This is an open challenge. Provide me with any evidence that disproves round earth and I'll give an explanation using mathematics and physics on why it is wrong.

The only assumption I am going to make is that the mathematics and physics I have been taught is true.

Serious one - At high alt, right on boundary of space.

What stops small particles escaping? they must be colliding, moving about so whats stopping them just moving that little bit more over that line and floating off?

Could that be happening but they are getting replaced by inbound particles kind of a constant balancing act ?

Mars got me thinking about this.

It's a combination of various things: Gravity, the Earth's magnetic field and the cycles of the dynamic atmosphere.
Watch the flat earth proponents telling me that any source I post is part of the great conspiracy.
However, for those serious about debate:
https://en.wikipedia.org/wiki/Solar_wind#Atmospheres
http://sciencing.com/earths-magnetosphere-protects-suns-solar-wind-1955.html - this one actually talks about Mars too
https://en.wikipedia.org/wiki/Atmospheric_escape#Dominant_atmospheric_escape_and_loss_processes_on_Earth

Yes, I'm using wikipedia - it's laid out concisely and simply and if you want more information there are countless links to follow. But since I ask more of people coming at this from the other side, I'm also going to explain how I understand what's going on:

The Earth's gravity keeps the atmosphere close to it, just like it does with the moon and the plethora of space junk accruing in orbit - much of which can be observed with nothing more than a pair of binoculars if you know where to look. On top of that, the dynamic centre of the earth produces a magnetic field that keeps us safe from the ravages of the near-vacuum of space. Hot metal spinning fast produces electromagnetic currents - this is a scientific fact you can verify yourself if you really want to (but this isn't a discussion of electrodynamics - go to a physics board if you want to argue that.) Plus, anyone who has used a compass has observed the effect of the Earth's magnetic field.
The most commonly observable consequences of this solar wind hitting the magnetosphere are the aurorae which happen above both the north and the south pole. This does send charged particles into the atmosphere, but this isn't where we get the matter back. Most of the air lost to the solar wind is replenished by the Earth itself. On top of that, oxygen gas is too massive to be stripped away by the solar wind quickly thanks to a combination of the magnetosphere and Earth's gravity. Over astronomical time periods, the atmosphere is being stripped away but due to the size of the Earth, this effect is negligible on human time scales.

Incidentally, in the Flat Earth model, those aurorae should be visible from everywhere on Earth. This is not the case.

tl;dr - the particles do escape, but it's happening really slowly because the Earth is really big and has a magnetic field

Ok so this is the kind of theory that stripped mars a slow decay over time. You did not say if you thought planets could exchange matter in both direction, (billions of mini meteorites, etc entering/exiting) 

I wonder if we are having a mass migration of matter from outer planets to inner planets (suns gravity) we could fix mars if say something smashes into an outer planet freeing a heap of ice, etc. 
When mars was forming it was going through an ACCUMULATION stage, then at some point it started to reverse, it could happen again. :)

OR is the solar wind pushing everything out ? 

Title: Re: [Ask Me Anything] Will explain every observation using round earth model
Post by: Yashas on April 04, 2017, 08:09:13 PM
Re: [Ask Me Anything] Will explain every observation using round earth model

Rotational speed of outer section of galaxies?

Irrelevant to FE or RE models. I could still answer your question but I am not able to understand what you are asking.
Title: Re: [Ask Me Anything] Will explain every observation using round earth model
Post by: Yashas on April 04, 2017, 08:15:33 PM
This is an open challenge. Provide me with any evidence that disproves round earth and I'll give an explanation using mathematics and physics on why it is wrong.

The only assumption I am going to make is that the mathematics and physics I have been taught is true.

Why is there a waterfall on Lake Victoria, Africa, where the said centrifugal forces are bulging the water out to make the earth globe? If the earth's rotation is bulging the water out at the equator, how can it also fall to a lower spot on earth? If the water at the equator is being bulged out to make the globe, why is the large Lake Victoria, sitting at the equator, so level and horizontal across its surface?

http://vizts.com/wp-content/uploads/2016/04/Victoria-Falls-amazing-view.jpg

I have already told you before. If you are ignorant and don't make an attempt to understand, I can't help you.

(http://image.prntscr.com/image/b29e75941a4e4d599c09853f2d2fee8d.png)

In fact, you weigh less near the equator but this difference is negligible. Over large scales, the bulge is measurable. It is 26 miles. Still small though. It would be a few pixels wide in a photo of earth from space. Any claim which tells the earth looks like a perfect sphere in NASA's photons is by people who don't understand this simple fact.

/Topic Closed

If you are mathematically disabled, here is an intuitive explanation: The wheel of your vehicle is rotating extremely fast. Why doesn't the rim fly away? It doesn't because it is a rigid body and there are internal forces which keep it together. These internal forces are more powerful than the centrifugal force. If you spin it too fast, you can actually break the wheel apart.

In our case, the gravity is holding the atmosphere together against the centrifugal force. As you can see in my calculations, the centrifugal force is too week. It does cause a bulge but it isn't significant. The gravitational force is much stronger and it keeps the water towards itself.

Variation of the earth's gravity with latitude: https://en.wikipedia.org/wiki/Gravity_of_Earth#Latitude

Flat earth model cannot explain the above. I won't bother bringing it in here because I don't expect FEers to have a serious mathematical and physics background. There are many other simple things which FE cannot explain.
Title: Re: [Ask Me Anything] Will explain every observation using round earth model
Post by: disputeone on April 04, 2017, 08:19:54 PM
Things can experience more than one force at a time.
Title: Re: [Ask Me Anything] Will explain every observation using round earth model
Post by: Yashas on April 04, 2017, 08:29:23 PM
This is an open challenge. Provide me with any evidence that disproves round earth and I'll give an explanation using mathematics and physics on why it is wrong.

The only assumption I am going to make is that the mathematics and physics I have been taught is true.

Serious one - At high alt, right on boundary of space.

What stops small particles escaping? they must be colliding, moving about so whats stopping them just moving that little bit more over that line and floating off?

Could that be happening but they are getting replaced by inbound particles kind of a constant balancing act ?

Mars got me thinking about this.

We are losing our atmosphere for many reasons. Mainly, solar flares scrape away the upper part of the atmosphere often. Secondly, there are few air molecules which move faster than the escape velocity.

The speeds of the molecules are stastically governed by Maxwell-Boltzmann equation. Given below is a plot known as Maxwell Distribution Curve.

(http://ef.engr.utk.edu/hyperphysics/hbase/Kinetic/imgkin/mxspd.gif)

X-axis = speed
Y-axis = probability of a gas having that speed
Area under the curve from [a,b] = total number of molecules having speed between [a,b]

The above picture gives you the total number of molecules with speed <X> . The area under any section of the curve gives you the number of molecules.

While the majority of the molecules have speeds which lie in the center, there are few (tiny bit of them) which have very high velocities. These escape the earth every day.

A tiny fraction of the gasses escape. While there are processes which remove the gas from the atmosphere, there are processes which add new gas to the atmosphere: meteorites? There are 60,000 tons of material added to the earth from meteorites every year. Most of these get burnt up in the atmosphere due to friction (they are traveling pretty fast and they heat up as they decelerate through the atmosphere). Some of them don't get burnt up and they hit the ground creating large craters.

Russian Meteor Explosion:

Our estimates indicate that it would take trillions of years for all the hydrogen in the atmosphere to be depleted. Why am I talking about hydrogen instead of the air as a whole? Hydrogen is the lightest element and it has the highest chance of leaving the earth. The velocity of the air molecules is a function of the temperature and molecular mass. If hydrogen depletes slowly, then other gasses are going to deplete even slower.

If you were on a more silly note, the earth gravity keeps the air together. The air is more dense near the surface and its density decreases as you go higher. The earth's gravity is strong enough to retain majority of its atmosphere.

Mars has a very dense atmosphere. Relatively lighter elements such as CO2 are thought to be removed by the solar winds. Work is still in progress. There are a dozen rovers on the surface of the Mars currently investigating. We don't know everything about Mars yet.

Atmospheric escape: https://en.wikipedia.org/wiki/Atmospheric_escape
Maxwell's Distribution: http://ef.engr.utk.edu/hyperphysics/hbase/Kinetic/kintem.html
We lose 50,000 tons of mass every year (good read): https://scitechdaily.com/earth-loses-50000-tonnes-of-mass-every-year/
Title: Re: [Ask Me Anything] Will explain every observation using round earth model
Post by: JackBlack on April 05, 2017, 12:37:47 AM
Serious one - At high alt, right on boundary of space.

What stops small particles escaping? they must be colliding, moving about so whats stopping them just moving that little bit more over that line and floating off?

Could that be happening but they are getting replaced by inbound particles kind of a constant balancing act ?

Mars got me thinking about this.
There is no sharp boundary of space, it just fades off.
As you get high enough, the gas begins to act as particles.
Some of it does fly off, some comes in from elsewhere. But the main factor is gravity. It is now acting as a particle, not as a gas. The vast majority of these particles are well below the escape velocity (for Earth anyway). This means they will go into various orbits, mostly elliptical orbits resulting in them colliding with the gas below them.

But yes, a small portion will be above the escape velocity and leave. For smaller objects, like the moon, the escape velocity is much smaller, allowing far more to leave.
Title: Re: [Ask Me Anything] Will explain every observation using round earth model
Post by: JackBlack on April 05, 2017, 12:38:49 AM
Re: [Ask Me Anything] Will explain every observation using round earth model

Rotational speed of outer section of galaxies?

Irrelevant to FE or RE models. I could still answer your question but I am not able to understand what you are asking.
He is asking why the outer sections of galaxies are rotating at a speed which doesn't match the amount of visible matter in the galaxy.
Title: Re: [Ask Me Anything] Will explain every observation using round earth model
Post by: Yashas on April 05, 2017, 12:47:26 AM
Serious one - At high alt, right on boundary of space.

What stops small particles escaping? they must be colliding, moving about so whats stopping them just moving that little bit more over that line and floating off?

Could that be happening but they are getting replaced by inbound particles kind of a constant balancing act ?

Mars got me thinking about this.
There is no sharp boundary of space, it just fades off.
As you get high enough, the gas begins to act as particles.
Some of it does fly off, some comes in from elsewhere. But the main factor is gravity. It is now acting as a particle, not as a gas. The vast majority of these particles are well below the escape velocity (for Earth anyway). This means they will go into various orbits, mostly elliptical orbits resulting in them colliding with the gas below them.

But yes, a small portion will be above the escape velocity and leave. For smaller objects, like the moon, the escape velocity is much smaller, allowing far more to leave.

Gases are made up of particles? So are liquids and solids?

Good point about the Moon. The moon does not have an atmosphere because its escape velocity is 5 times lesser than that of earth. The moon's gravity isn't powerful enough to retain an atmosphere. The average surface temperature of the moon (100 C) is higher than that of earth (16 C).

Re: [Ask Me Anything] Will explain every observation using round earth model

Rotational speed of outer section of galaxies?

Irrelevant to FE or RE models. I could still answer your question but I am not able to understand what you are asking.
He is asking why the outer sections of galaxies are rotating at a speed which doesn't match the amount of visible matter in the galaxy.

That is an open question. Answer it and you'll win the next nobel prize.

Many hypotheses have been put forward to explain the observations but none have been confirmed. The most popular hypothesis is dark matter.

https://en.wikipedia.org/wiki/Galaxy_rotation_curve

This question is irrelevant to the FE-RE discussion. I was asking for observations which FE explains or which debunks RE. Does FE have a solution for this? :P
Title: Re: [Ask Me Anything] Will explain every observation using round earth model
Post by: disputeone on April 05, 2017, 01:44:37 AM
You said: Ask any question and I will answer it with the round earth model.

Well we can't really explain the rotational speed of galaxies.

Our knowlege of GR and centrifugal force doesn't adequately explain it. We need to evoke dark matter which is a cool hypothesis but not yet proven to be a theory.

Because GR works so well on a small scale it's hard to say that it's "wrong" however maybe an incomplete understanding would be a good phrase.

A big gripe of mine is that new RE posters come here acting like we understand the entire universe. We don't, I very much doubt we understand 0.1% of it.

We do have some pretty sweet theories that explain observations and can make accurate predictions.
Title: Re: [Ask Me Anything] Will explain every observation using round earth model
Post by: Yashas on April 05, 2017, 01:52:27 AM
Who said RE people understand everything of the universe? FE does not explain this either and it will never will because its foundation itself is wrong. Well, it isn't even a scientific theory. It does not make predictions nor does it explain.

To anyone with enough commonsense, it is understood that I am referring to observations which are are related to flat and round earth. This had nothing to do with RE or FE.

"We do have some pretty sweet theories that explain observations and can make accurate predictions."
That is what I am looking for. Quote those and I'll tell you why it is wrong using mainstream science and mathematics. I don't invoke bullshit such as accelerating earth or atmosphere stringency or whatever.

Any observation which has nothing to do with earth being round or flat is irrelevant to this discussion.
Title: Re: [Ask Me Anything] Will explain every observation using round earth model
Post by: Bullwinkle on April 05, 2017, 02:34:54 AM
I very much doubt we understand 0.1% of it.

And the more we learn, that number will not change much.

Someone once said, (something like), "understanding the intricacies the atom is like dropping a grand piano down a flight of stairs and understanding Beethoven from the pieces."







Title: Re: [Ask Me Anything] Will explain every observation using round earth model
Post by: physical observer on April 05, 2017, 03:49:15 AM
This is an open challenge. Provide me with any evidence that disproves round earth and I'll give an explanation using mathematics and physics on why it is wrong.

The only assumption I am going to make is that the mathematics and physics I have been taught is true.

Why is there a waterfall on Lake Victoria, Africa, where the said centrifugal forces are bulging the water out to make the earth globe? If the earth's rotation is bulging the water out at the equator, how can it also fall to a lower spot on earth? If the water at the equator is being bulged out to make the globe, why is the large Lake Victoria, sitting at the equator, so level and horizontal across its surface?

http://vizts.com/wp-content/uploads/2016/04/Victoria-Falls-amazing-view.jpg

Lake Victoria is subject to gravity AND centrifugal force.
We keep coming back to this - why do you think physical forces are mutually exclusive?

And while were at it, have you been there?

Now let's use your reasoning:
How do we know that's a picture of Lake Victoria? We have to take your word for it. There is no way for us to verify that that is what you say it is.
Just another empty assertion with fake pictures.

If you want to use photographic evidence to support your claims, you can't discount any of the photos that have been used to discount yours.

"Lake Victoria is subject to gravity AND centrifugal force."

Water can't obey both forces at the same time. Water in this bucket is being controlled by centrifugal forces, notice none of the water is trying to fall to a lower point on earth, it is being forced away from the container that holds it, the bucket. Water cannot be controlled by both forces at the same time. 



You said you could disprove the evidence for flat earth with math and physics, all you could do was dismiss a picture of LV as being an actual photo. Go figure, another failure by an RE-ers. Typical!
Title: Re: [Ask Me Anything] Will explain every observation using round earth model
Post by: JackBlack on April 05, 2017, 04:06:48 AM
Water can't obey both forces at the same time. Water in this bucket is being controlled by centrifugal forces, notice none of the water is trying to fall to a lower point on earth, it is being forced away from earth. Water cannot be controlled by both forces at the same time.
Yes, it can.
It can't be dominated by both.

Planes fly by a balance of a multitude of forces.
When it increases the lift on one wing to bank right, it doesn't magically just get controlled by that one force.

Do you have a rational objection, or can you just repeat the same nonsense?

You said you could disprove the evidence for flat earth with math and physics, all you could do was dismiss a picture of LV as being an actual photo. Go figure, another failure by an RE-ers. Typical!
No. I pointed out why your claim is false, and that water will obey both forces, and that results int the equator bulging.
What was your refutation? Just repeating the same refuted crap.

So no, another fail by you FE-ers.
Title: Re: [Ask Me Anything] Will explain every observation using round earth model
Post by: physical observer on April 05, 2017, 05:50:44 AM
Water can't obey both forces at the same time. Water in this bucket is being controlled by centrifugal forces, notice none of the water is trying to fall to a lower point on earth, it is being forced away from earth. Water cannot be controlled by both forces at the same time.
Yes, it can.
It can't be dominated by both.

Planes fly by a balance of a multitude of forces.
When it increases the lift on one wing to bank right, it doesn't magically just get controlled by that one force.

Do you have a rational objection, or can you just repeat the same nonsense?

You said you could disprove the evidence for flat earth with math and physics, all you could do was dismiss a picture of LV as being an actual photo. Go figure, another failure by an RE-ers. Typical!
No. I pointed out why your claim is false, and that water will obey both forces, and that results int the equator bulging.
What was your refutation? Just repeating the same refuted crap.

So no, another fail by you FE-ers.

"It can't be dominated by both."

I see you offered no evidence to support your claim. I offered visual evidence to support my claim. You are the one with the crap! Fact, you are not even good enough to have any crap. In order to have crap, you have to ingest something of substance. Your Mickey Mouse intelligence doesn't register on the intelligence scale of substance.
Title: Re: [Ask Me Anything] Will explain every observation using round earth model
Post by: physical observer on April 05, 2017, 06:05:16 AM
Serious one - At high alt, right on boundary of space.

What stops small particles escaping? they must be colliding, moving about so whats stopping them just moving that little bit more over that line and floating off?

Could that be happening but they are getting replaced by inbound particles kind of a constant balancing act ?

Mars got me thinking about this.
There is no sharp boundary of space, it just fades off.
As you get high enough, the gas begins to act as particles.
Some of it does fly off, some comes in from elsewhere. But the main factor is gravity. It is now acting as a particle, not as a gas. The vast majority of these particles are well below the escape velocity (for Earth anyway). This means they will go into various orbits, mostly elliptical orbits resulting in them colliding with the gas below them.

But yes, a small portion will be above the escape velocity and leave. For smaller objects, like the moon, the escape velocity is much smaller, allowing far more to leave.

Gases are made up of particles? So are liquids and solids?

Good point about the Moon. The moon does not have an atmosphere because its escape velocity is 5 times lesser than that of earth. The moon's gravity isn't powerful enough to retain an atmosphere. The average surface temperature of the moon (100 C) is higher than that of earth (16 C).

Re: [Ask Me Anything] Will explain every observation using round earth model

Rotational speed of outer section of galaxies?

Irrelevant to FE or RE models. I could still answer your question but I am not able to understand what you are asking.
He is asking why the outer sections of galaxies are rotating at a speed which doesn't match the amount of visible matter in the galaxy.

That is an open question. Answer it and you'll win the next nobel prize.

Many hypotheses have been put forward to explain the observations but none have been confirmed. The most popular hypothesis is dark matter.

https://en.wikipedia.org/wiki/Galaxy_rotation_curve

This question is irrelevant to the FE-RE discussion. I was asking for observations which FE explains or which debunks RE. Does FE have a solution for this? :P

"The most popular hypothesis is dark matter."

Thus it becomes the most popular and accepted explanation in the realm of materialistic mainstream science. Mentally being put on a pedestal with a law/principle of science. Like the rest of the institutions of this world, science has become a total abortion of truth.

"Does FE have a solution for this? :P"

Didn't you claim to be the one explaining everything?

Title: Re: [Ask Me Anything] Will explain every observation using round earth model
Post by: Yashas on April 05, 2017, 06:36:34 AM
Water can't obey both forces at the same time. Water in this bucket is being controlled by centrifugal forces, notice none of the water is trying to fall to a lower point on earth, it is being forced away from the container that holds it, the bucket. Water cannot be controlled by both forces at the same time. 

You are standing on the ground because the floor is exerting a normal force upwards to prevent the gravitational force from taking you down. There are forces due to air resistance acting at the same time.

The fact that you told that two forces cannot act together indicates that you have probably failed 8th grade.
Title: Re: [Ask Me Anything] Will explain every observation using round earth model
Post by: Novarus on April 05, 2017, 12:32:36 PM
Serious one - At high alt, right on boundary of space.

What stops small particles escaping? they must be colliding, moving about so whats stopping them just moving that little bit more over that line and floating off?

Could that be happening but they are getting replaced by inbound particles kind of a constant balancing act ?

Mars got me thinking about this.
There is no sharp boundary of space, it just fades off.
As you get high enough, the gas begins to act as particles.
Some of it does fly off, some comes in from elsewhere. But the main factor is gravity. It is now acting as a particle, not as a gas. The vast majority of these particles are well below the escape velocity (for Earth anyway). This means they will go into various orbits, mostly elliptical orbits resulting in them colliding with the gas below them.

But yes, a small portion will be above the escape velocity and leave. For smaller objects, like the moon, the escape velocity is much smaller, allowing far more to leave.

Gases are made up of particles? So are liquids and solids?

Good point about the Moon. The moon does not have an atmosphere because its escape velocity is 5 times lesser than that of earth. The moon's gravity isn't powerful enough to retain an atmosphere. The average surface temperature of the moon (100 C) is higher than that of earth (16 C).

Re: [Ask Me Anything] Will explain every observation using round earth model

Rotational speed of outer section of galaxies?

Irrelevant to FE or RE models. I could still answer your question but I am not able to understand what you are asking.
He is asking why the outer sections of galaxies are rotating at a speed which doesn't match the amount of visible matter in the galaxy.

That is an open question. Answer it and you'll win the next nobel prize.

Many hypotheses have been put forward to explain the observations but none have been confirmed. The most popular hypothesis is dark matter.

https://en.wikipedia.org/wiki/Galaxy_rotation_curve

This question is irrelevant to the FE-RE discussion. I was asking for observations which FE explains or which debunks RE. Does FE have a solution for this? :P

"The most popular hypothesis is dark matter."

Thus it becomes the most popular and accepted explanation in the realm of materialistic mainstream science. Mentally being put on a pedestal with a law/principle of science. Like the rest of the institutions of this world, science has become a total abortion of truth.

"Does FE have a solution for this? :P"

Didn't you claim to be the one explaining everything?

The platform is yours if you want to explain it, physical observer.

Or you can do what any sensible scientist who doesn't know does:

Say "I don't know."

Then maybe yes can get back to the lost of questions you haven't answered.


You can take a crack, Yashas - I can guarantee you will answer them better.
Title: Re: [Ask Me Anything] Will explain every observation using round earth model
Post by: JackBlack on April 05, 2017, 02:34:29 PM
I see you offered no evidence to support your claim. I offered visual evidence to support my claim. You are the one with the crap! Fact, you are not even good enough to have any crap. In order to have crap, you have to ingest something of substance. Your Mickey Mouse intelligence doesn't register on the intelligence scale of substance.
I did provide evidence, in the forms of planes, which would fall out of the sky if they could only have one force acting upon them, as well as a multitude of other examples previously.

You didn't offer any evidence to support your claim.
Water in a bucket doesn't show that multiple forces can't act at once.

Perhaps this is a simpler one for you to understand:
Is a broken down car easier to move if you have multiple people pushing it?
See, if the BS you are saying is true, then only one force can act on the car at a given time.
So if someone stars pushing it, in order for that force to have an effect, all other forces can't be acting on it, that means no gravity (or whatever bullshit you want in its place) holding it down so it should start just floating away.
If someone else starts pushing it, well you can only have 1 force acting at once, so that means for them to have an effect they can be the only one providing any force to it, so the other people pushing it are useless.

Back in reality, instead of your delusional fantasy world, multiple forces can act at once.
That means each person is providing a force to the car trying to move it and all those forces are acting on the car. This is why multiple people can push something (or have it easier to do) when one of them couldn't do it alone (or would find it very difficult), and gravity continues acting on it holding the car down.

Do you have any actual evidence to support your claim, or just bullshit?

Are you aware that several of the videos you have provided previously refute your claim and instead show exactly what you are arguing against now?

Here one is:


If what you are saying is true, then the water must either obey gravity (or whatever bullshit you want in its place), and fall to the lowest point, becoming a level surface, or it must obey centrifugal forces, and be thrown to the sides of the container.

But what actually happens?
It obeys both.
There is the gravitational force pulling it down, but there is also the centrifugal force pushing it to the side. This results in it bulging up at the sides and the surface being in equilibrium with the forces balanced.

Is that enough evidence for you?
Or do you have an explanation of what is going on in those videos which doesn't involve 2 or more forces?
Title: Re: [Ask Me Anything] Will explain every observation using round earth model
Post by: physical observer on April 05, 2017, 03:03:59 PM
Water can't obey both forces at the same time. Water in this bucket is being controlled by centrifugal forces, notice none of the water is trying to fall to a lower point on earth, it is being forced away from the container that holds it, the bucket. Water cannot be controlled by both forces at the same time. 

You are standing on the ground because the floor is exerting a normal force upwards to prevent the gravitational force from taking you down. There are forces due to air resistance acting at the same time.

The fact that you told that two forces cannot act together indicates that you have probably failed 8th grade.

One more unsupported, unprovable sorry excuse. In the longggggggggggg line of sorry ass excuses. If gravity and centrifugal forces can act on water at the same time, then why doesn't the water in the rotating bucket fall to the lowest place on earth? The centrifugal force of earth's spin is said to be strong enough to force the water on earth into a bulge out, and even more at the equator. Lake Victoria, right on the equator, shows no signs of bulging out over its surface. That large body of water is flat and horizontal to plane earth, even boasting a waterfall, where the water is seeking the next lowest point it can fall to, it is not bulging out.

Two opposite forces, gravity, and centrifugal force, cannot act on water at the same time. Saying I flunked 8th grade is an insult, because you know you're wrong about Lake Victoria and the centrifugal force of the alleged spinning globe. The water in Lake Victoria behaves like it is on a motionless plane, not a spinning speeding ball.

Only a person with a defunct argument needs to hurl insults at someone with a different idea. Especially if the idea is beyond refutation. You can't refute what earth's nature shows:



Tell you what, Africa looks pretty dang flat, and that large body of water does not behave like centrifugal forces are bulging out. You can believe what you wish, but the evidence from earth's physical state, just does not support a spinning globe.
Title: Re: [Ask Me Anything] Will explain every observation using round earth model
Post by: Novarus on April 05, 2017, 03:11:27 PM
Water can't obey both forces at the same time. Water in this bucket is being controlled by centrifugal forces, notice none of the water is trying to fall to a lower point on earth, it is being forced away from the container that holds it, the bucket. Water cannot be controlled by both forces at the same time. 

You are standing on the ground because the floor is exerting a normal force upwards to prevent the gravitational force from taking you down. There are forces due to air resistance acting at the same time.

The fact that you told that two forces cannot act together indicates that you have probably failed 8th grade.

One more unsupported, unprovable sorry excuse. In the longggggggggggg line of sorry ass excuses. If gravity and centrifugal forces can act on water at the same time, then why doesn't the water in the rotating bucket fall to the lowest place on earth? The centrifugal force of earth's spin is said to be strong enough to force the water on earth into a bulge out, and even more at the equator. Lake Victoria, right on the equator, shows no signs of bulging out over its surface. That large body of water is flat and horizontal to plane earth, even boasting a waterfall, where the water is seeking the next lowest point it can fall to, it is not bulging out.

Two opposite forces, gravity, and centrifugal force, cannot act on water at the same time. Saying I flunked 8th grade is an insult, because you know you're wrong about Lake Victoria and the centrifugal force of the alleged spinning globe. The water in Lake Victoria behaves like it is on a motionless plane, not a spinning speeding ball.

Only a person with a defunct argument needs to hurl insults at someone with a different idea. Especially if the idea is beyond refutation. You can't refute what earth's nature shows:



Tell you what, Africa looks pretty dang flat, and that large body of water does not behave like centrifugal forces are bulging out. You can believe what you wish, but the evidence from earth's physical state, just does not support a spinning globe.

Once again, physical forces are not mutually exclusive.

And sorry to bring this up again, but if you don't accept our photographic evidence, how can you use photos and videos to support yours?

(Excuse me while I get flat for a sec)
How can we take your word that this is Victoria Falls? How do we know this isn't fake?
Have you been there yourself?  Have you observed this physically?
Title: Re: [Ask Me Anything] Will explain every observation using round earth model
Post by: Bullwinkle on April 05, 2017, 03:28:27 PM

(http://)



Notice the fluid surface forms a parabolic shape?

Thought that was cool.

Carry on . . . . .
Title: Re: [Ask Me Anything] Will explain every observation using round earth model
Post by: JackBlack on April 05, 2017, 04:09:52 PM
One more unsupported, unprovable sorry excuse.
No. One more solid rebuttal against your BS.
It is a situation where 2 opposing forces are acting upon you.

If gravity and centrifugal forces can act on water at the same time, then why doesn't the water in the rotating bucket fall to the lowest place on earth?
Because in this situation the centrifugal forces are greater, causing it to fly to the outside of the bucket. If you slow the bucket down enough, eventually gravity wins and you get wet.

The centrifugal force of earth's spin is said to be strong enough to force the water on earth into a bulge out, and even more at the equator.
Yes, a very tiny amount. The bulge is a mere 0.15%.


Lake Victoria, right on the equator, shows no signs of bulging out over its surface.
That is because you aren't viewing a large enough area.
Even if you were viewing all of Earth, the bulge is so small you cannot detect without a guide or instrument.


That large body of water is flat and horizontal to plane earth
Except you can't show it is flat.
There is no indication it isn't following the curve of Earth.


where the water is seeking the next lowest point it can fall to, it is not bulging out.
Again, you just can't detect it.

Not detecting it doesn't mean it isn't there.

Two opposite forces, gravity, and centrifugal force, cannot act on water at the same time.
Yes, they can. They do so quite often.

You even provided a video where it happened.
Here is that video again:


If only one force could act then either the water would have to fall to the bottom of the container, being completely consistent with a stationary one, or it would need to fly to the side, not being under the force of gravity (or whatever else you want to say makes things fall) at all.

Instead, the water surface is at an angle, in one case following a parabola (at least approximately).

If you wish to assert they cannot act at the same time you will need to provide a reason (and preferably evidence) which indicates they can't.

Having one force massively larger than the other so it appears that only one is acting is not demonstrating the other one is not acting, all it does is indicate the other is too small to detect in that experiment.

You would need a situation where the 2 forces are comparable in magnitude (like say the video you provided), where only one acts.


because you know you're wrong about Lake Victoria and the centrifugal force of the alleged spinning globe. The water in Lake Victoria behaves like it is on a motionless plane, not a spinning speeding ball.
No. We know you are wrong.
On the small scale, the water in Lake Victoria behaves like Earth was either a motionless plane or a spinning, speeding ball.
There is no way to distinguish them.
On the larger scale, it acts like a spinning ball, not a motionless plane.

Especially if the idea is beyond refutation. You can't refute what earth's nature shows:
Good think your idea is very easy to refute and goes completely against what Earth's nature shows.


Tell you what, Africa looks pretty dang flat, and that large body of water does not behave like centrifugal forces are bulging out.
It looks pretty consistent with both, until you get a curved horizon.

You can believe what you wish, but the evidence from earth's physical state, just does not support a spinning globe.
No. It does support it.
Title: Re: [Ask Me Anything] Will explain every observation using round earth model
Post by: JackBlack on April 05, 2017, 04:11:20 PM
Notice the fluid surface forms a parabolic shape?

Thought that was cool.

Carry on . . . . .
It looks much cooler when you have a parabolic dish with a lip).
You put the water in, then get it spinning at just the right speed and it has a nice thin coating over the entire surface.
Title: Re: [Ask Me Anything] Will explain every observation using round earth model
Post by: Novarus on April 05, 2017, 06:20:13 PM
Notice the fluid surface forms a parabolic shape?

Thought that was cool.

Carry on . . . . .
It looks much cooler when you have a parabolic dish with a lip).
You put the water in, then get it spinning at just the right speed and it has a nice thin coating over the entire surface.

What about a Flat dish accelerating at 9.8m/s/s?
Genuinely curious as to how this would look
Title: Re: [Ask Me Anything] Will explain every observation using round earth model
Post by: Yashas on April 05, 2017, 06:23:38 PM
I won't bother answering because I have already answered. If physical observer cannot understand high school level physics and math, I am helpless.

@Physical Observer

How many forces are acting if two people are pulling you from two sides?
Title: Re: [Ask Me Anything] Will explain every observation using round earth model
Post by: rabinoz on April 05, 2017, 06:43:25 PM
Two opposite forces, gravity, and centrifugal force, cannot act on water at the same time.
Rubbish!

Spinning Liquid and Centripetal Force, PhysicsandAstronomy UKY
In that video,
It is the two forces, gravity, and centrifugal force acting together that makes the surface take on a parabolic shape.

Even if the two forces are opposing, the nett force is still just the vector sum.

At the equator
          the gravitational acceleration alone is about 9.84 m/s2 and
          the centrifugal acceleration alone is about 0.03 m/s2, so
the nett gravity (as we call it) is about 9.81 m/s2. I said "about" because it does depend on altitude and other factors.

In other words, the effect of the earth's rotation is only about 0.3% that of gravitation.

Is that too much to swallow?
Title: Re: [Ask Me Anything] Will explain every observation using round earth model
Post by: Zaphod on April 06, 2017, 02:10:32 AM
Physical Observer

I'm sorry but you are displaying astonishing scientific illiteracy. Of course multiple forces can act on an object at the same time. Does gravity just say "ok centrifugal force, your turn - I'll just nip out for a smoke and a pancake whilst you carry on"?

If you pick something up off the floor gravity doesn't look the other way for a bit. You just apply a greater force. The net vector addition has the resultant going up so the object moves upwards.

To be honest if you don't get this then there's pretty much no point debating anything with you.

Newton's laws are clearly laid out here...

https://en.m.wikipedia.org/wiki/Newton%27s_laws_of_motion
Title: Re: [Ask Me Anything] Will explain every observation using round earth model
Post by: physical observer on April 06, 2017, 02:38:16 AM
Physical Observer

I'm sorry but you are displaying astonishing scientific illiteracy. Of course multiple forces can act on an object at the same time. Does gravity just say "ok centrifugal force, your turn - I'll just nip out for a smoke and a pancake whilst you carry on"?

If you pick something up off the floor gravity doesn't look the other way for a bit. You just apply a greater force. The net vector addition has the resultant going up so the object moves upwards.

To be honest if you don't get this then there's pretty much no point debating anything with you.

Newton's laws are clearly laid out here...

https://en.m.wikipedia.org/wiki/Newton%27s_laws_of_motion

Number one, never, ever give me wiki as a source. Once I found wiki change the Principle of Biogenesis to the hypothesis of biogenesis, I banned them from my list of a respectable and acceptable information resource.

Number two, show me an case from earth's nature where water is obeying both forces, centrifugal and gravity, at the same time, BET YOU CAN'T!!!!!!!!
Title: Re: [Ask Me Anything] Will explain every observation using round earth model
Post by: Yashas on April 06, 2017, 02:45:49 AM
Physical Observer

I'm sorry but you are displaying astonishing scientific illiteracy. Of course multiple forces can act on an object at the same time. Does gravity just say "ok centrifugal force, your turn - I'll just nip out for a smoke and a pancake whilst you carry on"?

If you pick something up off the floor gravity doesn't look the other way for a bit. You just apply a greater force. The net vector addition has the resultant going up so the object moves upwards.

To be honest if you don't get this then there's pretty much no point debating anything with you.

Newton's laws are clearly laid out here...

https://en.m.wikipedia.org/wiki/Newton%27s_laws_of_motion

Number one, never, ever give me wiki as a source. Once I found wiki change the Principle of Biogenesis to the hypothesis of biogenesis, I banned them from my list of a respectable and acceptable information resource.

Number two, show me an case from earth's nature where water is obeying both forces, centrifugal and gravity, at the same time, BET YOU CAN'T!!!!!!!!

You are unbelievably scientifically illiterate. Even a 4-year-old kid understands that you can pull a person using both his hands. You are applying two forces.

There are millions (if not billions) of examples where more than one force is acting. Students from 9th grade to 12th grade solve hundreds of problems involving multiple forces.

Take a 9th grade textbook and start studying. You seem to have failed high school (if not primary school).
Title: Re: [Ask Me Anything] Will explain every observation using round earth model
Post by: physical observer on April 06, 2017, 02:57:39 AM
Physical Observer

I'm sorry but you are displaying astonishing scientific illiteracy. Of course multiple forces can act on an object at the same time. Does gravity just say "ok centrifugal force, your turn - I'll just nip out for a smoke and a pancake whilst you carry on"?

If you pick something up off the floor gravity doesn't look the other way for a bit. You just apply a greater force. The net vector addition has the resultant going up so the object moves upwards.

To be honest if you don't get this then there's pretty much no point debating anything with you.

Newton's laws are clearly laid out here...

https://en.m.wikipedia.org/wiki/Newton%27s_laws_of_motion

Number one, never, ever give me wiki as a source. Once I found wiki change the Principle of Biogenesis to the hypothesis of biogenesis, I banned them from my list of a respectable and acceptable information resource.

Number two, show me an case from earth's nature where water is obeying both forces, centrifugal and gravity, at the same time, BET YOU CAN'T!!!!!!!!

You are unbelievably scientifically illiterate. Even a 4-year-old kid understands that you can pull a person using both his hands. You are applying two forces.

There are millions (if not billions) of examples where more than one force is acting. Students from 9th grade to 12th grade solve hundreds of problems involving multiple forces.

Take a 9th grade textbook and start studying. You seem to have failed high school (if not primary school).

We are talking about water, dude, not people. Please show me the evidence where water is obeying both forces, gravity and centrifugal, at the same time. Your claims mean absolutely nothing without visual evidence to back it up. I baked up my claim with visual evidence, why can't you?

The Amazon River right on the equator in S. America:



The river shows no indication it is being acted upon by centrifugal forces. Please notice the shelf of water between the two falls, horizontally flat to plane earth across its surface. Now please show me your visual evidence of water being pulled by centrifugal force, and also falling to the lowest point possible on earth. Time to put your money where your mouth is.........................
Title: Re: [Ask Me Anything] Will explain every observation using round earth model
Post by: Yashas on April 06, 2017, 04:07:41 AM
Lol, you just lost the battle and adjusted your question. Anyway here is a proof for water too:

It is a common high school problem to find the shape of the surface in a rotating body. The result turns out to be parabolic. We consider the gravitational force + centrifugal force together to obtain the answer.



The water does not go through the container because the container is pushing the water back in. You have 3 obvious forces (+ more hidden forces).

P.S: If you had a little better understanding of science, I would have screwed you long ago. Every particle of water is feeling a force from every other particle of water in its vicinity. I don't expect you to understand it so let us not talk about it.
Title: Re: [Ask Me Anything] Will explain every observation using round earth model
Post by: Zaphod on April 06, 2017, 04:22:03 AM
Objects don't "obey" forces, they are subject to forces and and move according to Newton's laws. Forces are vector quantities and can be summed accordingly.

So, the water at the equator is being pulled towards centre of the earth by gravity, AND being flung out by centrifugal force. As has been said countless times gravity is a lot stronger so the water doesn't fly off. Nor do you fly off.

Newton's laws don't change just because they are on Wikipedia. Look them up and try to understand them. You must surely have covered them at school?

Please try to understand or there is little point entering into a discussion.

http://www.physicsclassroom.com/Physics-Tutorial/Newton-s-Laws
Title: Re: [Ask Me Anything] Will explain every observation using round earth model
Post by: Canadabear on April 06, 2017, 04:43:13 AM


We are talking about water, dude, not people. Please show me the evidence where water is obeying both forces, gravity and centrifugal, at the same time. Your claims mean absolutely nothing without visual evidence to back it up. I baked up my claim with visual evidence, why can't you?

The Amazon River right on the equator in S. America:



The river shows no indication it is being acted upon by centrifugal forces. Please notice the shelf of water between the two falls, horizontally flat to plane earth across its surface. Now please show me your visual evidence of water being pulled by centrifugal force, and also falling to the lowest point possible on earth. Time to put your money where your mouth is.........................

do you know how much centrifugal force is acting on the water?
even if you do not believe that the earth is global, do the calculation.

here an example:

lets calculate the force on a human body:

centrifugal force F=m*v^2/r

m=100kg
v=1000 mph = 447 m/s
r=6371km = 6371000 m

F=100 kg * 447^2 m^2/s^2 / 6371000m

F=3.13 kg m/s^2

as we know kg m/s^2 is Newton

its a force of 3N

that can be seen as a weight of 0.3 kg

that is only 0.3% of the body weight.

Title: Re: [Ask Me Anything] Will explain every observation using round earth model
Post by: physical observer on April 06, 2017, 04:44:02 AM
Lol, you just lost the battle and adjusted your question. Anyway here is a proof for water too:

It is a common high school problem to find the shape of the surface in a rotating body. The result turns out to be parabolic. We consider the gravitational force + centrifugal force together to obtain the answer.



The water does not go through the container because the container is pushing the water back in. You have 3 obvious forces (+ more hidden forces).

P.S: If you had a little better understanding of science, I would have screwed you long ago. Every particle of water is feeling a force from every other particle of water in its vicinity. I don't expect you to understand it so let us not talk about it.

That is not visual evidence that water on earth is being pulled to make earth's curvature, and also falling to a lower place on earth at the same time. Water on earth does not act like the water in the rotating container. You can't find a body of water on earth that is both being pulled from earth by centrifugal force, and falling to a lower place on earth at the same time, can you? You can't even find a body of water on earth that is being pulled away from earth by centrifugal force, can you? That is why you need to insult and use lab experiments that do not apply.

Notice the water in the rotating container becomes concave when centrifugal forces are applied, the water surface does not convex to make an arch, a bow across its surface, does it? If you spun that container to match earth's 1,000 MPH surface, I bet the water would fly out the open top of the container. You'd have to put a lid on it to keep the liquid in the container. Bodies of water on earth don't have lids on them, do they?
Title: Re: [Ask Me Anything] Will explain every observation using round earth model
Post by: Canadabear on April 06, 2017, 04:54:29 AM
Lol, you just lost the battle and adjusted your question. Anyway here is a proof for water too:

It is a common high school problem to find the shape of the surface in a rotating body. The result turns out to be parabolic. We consider the gravitational force + centrifugal force together to obtain the answer.



The water does not go through the container because the container is pushing the water back in. You have 3 obvious forces (+ more hidden forces).

P.S: If you had a little better understanding of science, I would have screwed you long ago. Every particle of water is feeling a force from every other particle of water in its vicinity. I don't expect you to understand it so let us not talk about it.

That is not visual evidence that water on earth is being pulled to make earth's curvature, and also falling to a lower place on earth at the same time. Water on earth does not act like the water in the rotating container. You can't find a body of water on earth that is both being pulled from earth by centrifugal force, and falling to a lower place on earth at the same time, can you? You can't even find a body of water on earth that is being pulled away from earth by centrifugal force, can you? That is why you need to insult and use lab experiments that do not apply.

Notice the water in the rotating container becomes concave when centrifugal forces are applied, the water surface does not convex to make an arch, a bow across its surface, does it? If you spun that container to match earth's 1,000 MPH surface, I bet the water would fly out the open top of the container. You'd have to put a lid on it to keep the liquid in the container. Bodies of water on earth don't have lids on them, do they?

we can not "see" it because the effect is constant and very little, as i showed you in the post above.
Title: Re: [Ask Me Anything] Will explain every observation using round earth model
Post by: physical observer on April 06, 2017, 05:02:27 AM


We are talking about water, dude, not people. Please show me the evidence where water is obeying both forces, gravity and centrifugal, at the same time. Your claims mean absolutely nothing without visual evidence to back it up. I baked up my claim with visual evidence, why can't you?

The Amazon River right on the equator in S. America:



The river shows no indication it is being acted upon by centrifugal forces. Please notice the shelf of water between the two falls, horizontally flat to plane earth across its surface. Now please show me your visual evidence of water being pulled by centrifugal force, and also falling to the lowest point possible on earth. Time to put your money where your mouth is.........................

do you know how much centrifugal force is acting on the water?
even if you do not believe that the earth is global, do the calculation.

here an example:

lets calculate the force on a human body:

centrifugal force F=m*v^2/r

m=100kg
v=1000 mph = 447 m/s
r=6371km = 6371000 m

F=100 kg * 447^2 m^2/s^2 / 6371000m

F=3.13 kg m/s^2

as we know kg m/s^2 is Newton

its a force of 3N

that can be seen as a weight of 0.3 kg

that is only 0.3% of the body weight.

"do you know how much centrifugal force is acting on the water?"

Enough to allegedly bow water on earth to make this:



But water at the equator looks like this, horizontally flat to plane earth across its surface:



Ocean water Senegal, Africa, right on the equator. I don't see centrifugal forces at work:



Now please, if you have visual evidence to support your claim, present it, or be honest and say you have none.
Title: Re: [Ask Me Anything] Will explain every observation using round earth model
Post by: physical observer on April 06, 2017, 05:06:03 AM
Lol, you just lost the battle and adjusted your question. Anyway here is a proof for water too:

It is a common high school problem to find the shape of the surface in a rotating body. The result turns out to be parabolic. We consider the gravitational force + centrifugal force together to obtain the answer.



The water does not go through the container because the container is pushing the water back in. You have 3 obvious forces (+ more hidden forces).

P.S: If you had a little better understanding of science, I would have screwed you long ago. Every particle of water is feeling a force from every other particle of water in its vicinity. I don't expect you to understand it so let us not talk about it.

That is not visual evidence that water on earth is being pulled to make earth's curvature, and also falling to a lower place on earth at the same time. Water on earth does not act like the water in the rotating container. You can't find a body of water on earth that is both being pulled from earth by centrifugal force, and falling to a lower place on earth at the same time, can you? You can't even find a body of water on earth that is being pulled away from earth by centrifugal force, can you? That is why you need to insult and use lab experiments that do not apply.

Notice the water in the rotating container becomes concave when centrifugal forces are applied, the water surface does not convex to make an arch, a bow across its surface, does it? If you spun that container to match earth's 1,000 MPH surface, I bet the water would fly out the open top of the container. You'd have to put a lid on it to keep the liquid in the container. Bodies of water on earth don't have lids on them, do they?

we can not "see" it because the effect is constant and very little, as i showed you in the post above.

Yep, we cannot see it, because it is not happening. I have the visual proof, you do not.
Title: Re: [Ask Me Anything] Will explain every observation using round earth model
Post by: totallackey on April 06, 2017, 05:12:05 AM
Incidentally, in the Flat Earth model, those aurorae should be visible from everywhere on Earth. This is not the case.
I am sorry, but aurora have been observed everywhere on Earth.
Title: Re: [Ask Me Anything] Will explain every observation using round earth model
Post by: Yashas on April 06, 2017, 05:19:04 AM
Incidentally, in the Flat Earth model, those aurorae should be visible from everywhere on Earth. This is not the case.
I am sorry, but aurora have been observed everywhere on Earth.

I don't see it. It can't happen either. The magnetic field of the earth diverts the charged particles towards the poles. Therefore, you can see aurora near the poles.

Please do your homework and don't make baseless fake claims.
Title: Re: [Ask Me Anything] Will explain every observation using round earth model
Post by: rabinoz on April 06, 2017, 05:22:57 AM

that is only 0.3% of the body weight.

"do you know how much centrifugal force is acting on the water?"

Enough to allegedly bow water on earth to make this:


Centrifugal force does not cause water to "bend around the earth"!
All it does in make the equatorial diameter of the earth about 0.3% larger than the polar diameter.

Quote from: physical observer
But water at the equator looks like this, horizontally flat to plane earth across its surface:



Ocean water Senegal, Africa, right on the equator. I don't see centrifugal forces at work:



Now please, if you have visual evidence to support your claim, present it, or be honest and say you have none.
Of course you "don't see centrifugal forces at work", because all they do is slightly reduce the effective gravity!

You have not the slightest concept of "relative values".

Sure, I agree that "water at the equator looks like this, horizontally flat to plane earth across its surface".
Because as I and others have explained numerous times, on the Globe it should look flat, especially over small areas.

We tell you the same things over and over and you take absolutely no notice.
Please explain why you ignore the simple explanations you are given numerous times!
Title: Re: [Ask Me Anything] Will explain every observation using round earth model
Post by: physical observer on April 06, 2017, 05:28:06 AM
Incidentally, in the Flat Earth model, those aurorae should be visible from everywhere on Earth. This is not the case.
I am sorry, but aurora have been observed everywhere on Earth.

I've seen it from my mountain perch in Maine.
Title: Re: [Ask Me Anything] Will explain every observation using round earth model
Post by: Canadabear on April 06, 2017, 06:09:40 AM


Yep, we cannot see it, because it is not happening. I have the visual proof, you do not.

we all have visual and logic prove that it is happening.

you stated in one post you teach your baseball team to use the magnus effect.
can you see the effect directly? no you only see the result of it.

there we can see that you are unable to use simple logic.
Title: Re: [Ask Me Anything] Will explain every observation using round earth model
Post by: physical observer on April 06, 2017, 06:24:57 AM


Yep, we cannot see it, because it is not happening. I have the visual proof, you do not.

we all have visual and logic prove that it is happening.

you stated in one post you teach your baseball team to use the magnus effect.
can you see the effect directly? no you only see the result of it.

there we can see that you are unable to use simple logic.

I have not been shown any bodies of water that rest in an earthly vessel, like a lake, river, pond, or ocean, that is being subjected to centrifugal forces. I have shown you'll bodies of water at the equator, where the centrifugal forces are said to be the strongest, that do not match water being subjected to centrifugal forces. Water at the equator is flat, level and horizontal across its surface, it does not bow in a curve, and still fall to a lower spot on earth.
Title: Re: [Ask Me Anything] Will explain every observation using round earth model
Post by: Canadabear on April 06, 2017, 06:30:37 AM


Yep, we cannot see it, because it is not happening. I have the visual proof, you do not.

we all have visual and logic prove that it is happening.

you stated in one post you teach your baseball team to use the magnus effect.
can you see the effect directly? no you only see the result of it.

there we can see that you are unable to use simple logic.

I have not been shown any bodies of water that rest in an earthly vessel, like a lake, river, pond, or ocean, that is being subjected to centrifugal forces. I have shown you'll bodies of water at the equator, where the centrifugal forces are said to be the strongest, that do not match water being subjected to centrifugal forces. Water at the equator is flat, level and horizontal across its surface, it does not bow in a curve, and still fall to a lower spot on earth.

the centrifugal force acts perpendicular to the water surface therefore you do not see it directly.

and whats with the magnus effect do you have no answer for it or do you simply ignoring it now?
Title: Re: [Ask Me Anything] Will explain every observation using round earth model
Post by: Yashas on April 06, 2017, 07:01:26 AM
Incidentally, in the Flat Earth model, those aurorae should be visible from everywhere on Earth. This is not the case.
I am sorry, but aurora have been observed everywhere on Earth.

I've seen it from my mountain perch in Maine.

Visual evidence please :P
Title: Re: [Ask Me Anything] Will explain every observation using round earth model
Post by: totallackey on April 06, 2017, 08:56:25 AM
Incidentally, in the Flat Earth model, those aurorae should be visible from everywhere on Earth. This is not the case.
I am sorry, but aurora have been observed everywhere on Earth.

I don't see it. It can't happen either. The magnetic field of the earth diverts the charged particles towards the poles. Therefore, you can see aurora near the poles.

Please do your homework and don't make baseless fake claims.

I do not give two shakes whether or not you have seen it.

Aurora have been reported from all over the Earth at all different latitudes and longitudes.

You go do some research before you throw out more crap.
Title: Re: [Ask Me Anything] Will explain every observation using round earth model
Post by: Zaphod on April 06, 2017, 09:07:32 AM
Physical Observer

Are you being deliberately obtuse? It's been explained to you over and over again. The resultant force on the water at the equator is still down as the centrifugal force (acting in the opposite directition to gravity) is tiny.

All the waters you see ARE curved. It's just that it's impossible to see as the radius of curvature is 6400 kilometres!

I'm beginning to think that this just one big wind up.

Did you look up Newton's laws?
Title: Re: [Ask Me Anything] Will explain every observation using round earth model
Post by: Canadabear on April 06, 2017, 09:09:25 AM
Incidentally, in the Flat Earth model, those aurorae should be visible from everywhere on Earth. This is not the case.
I am sorry, but aurora have been observed everywhere on Earth.

I don't see it. It can't happen either. The magnetic field of the earth diverts the charged particles towards the poles. Therefore, you can see aurora near the poles.

Please do your homework and don't make baseless fake claims.

I do not give two shakes whether or not you have seen it.

Aurora have been reported from all over the Earth at all different latitudes and longitudes.

You go do some research before you throw out more crap.

was it always a Aurora or some other phenomenon?
can you give us some reports.

BTW
here

 https://en.wikipedia.org/wiki/Aurora,_Ontario

you can see Aurora at any time, like i did.  ;D
Title: Re: [Ask Me Anything] Will explain every observation using round earth model
Post by: Canadabear on April 06, 2017, 09:14:34 AM
Water can't obey both forces at the same time. Water in this bucket is being controlled by centrifugal forces, notice none of the water is trying to fall to a lower point on earth, it is being forced away from earth. Water cannot be controlled by both forces at the same time.
Yes, it can.
It can't be dominated by both.

Planes fly by a balance of a multitude of forces.
When it increases the lift on one wing to bank right, it doesn't magically just get controlled by that one force.

Do you have a rational objection, or can you just repeat the same nonsense?

You said you could disprove the evidence for flat earth with math and physics, all you could do was dismiss a picture of LV as being an actual photo. Go figure, another failure by an RE-ers. Typical!
No. I pointed out why your claim is false, and that water will obey both forces, and that results int the equator bulging.
What was your refutation? Just repeating the same refuted crap.

So no, another fail by you FE-ers.

"It can't be dominated by both."

I see you offered no evidence to support your claim. I offered visual evidence to support my claim. You are the one with the crap! Fact, you are not even good enough to have any crap. In order to have crap, you have to ingest something of substance. Your Mickey Mouse intelligence doesn't register on the intelligence scale of substance.

simply if you slow down your circle movement of the bucket the water will start to fall out of the bucket.
because the centrifugal force will get smaller and finally is smaller that the gravitational force.

a very simple experiment you can do yourself
Title: Re: [Ask Me Anything] Will explain every observation using round earth model
Post by: deadsirius on April 06, 2017, 09:15:50 AM

Physical Observer

Are you being deliberately obtuse?


Yes.
Title: Re: [Ask Me Anything] Will explain every observation using round earth model
Post by: physical observer on April 06, 2017, 09:30:04 AM
Physical Observer

Are you being deliberately obtuse? It's been explained to you over and over again. The resultant force on the water at the equator is still down as the centrifugal force (acting in the opposite directition to gravity) is tiny.

All the waters you see ARE curved. It's just that it's impossible to see as the radius of curvature is 6400 kilometres!

I'm beginning to think that this just one big wind up.

Did you look up Newton's laws?

None of you can show me the physical evidence form earth, can you? Impossible to see the curvature, can't find any bodies of water on earth dealing with centrifugal forces, the ground feels motionless.

"Did you look up Newton's laws?"

Did you look at the physical condition of earth?
Title: Re: [Ask Me Anything] Will explain every observation using round earth model
Post by: inquisitive on April 06, 2017, 09:44:02 AM
Physical Observer

Are you being deliberately obtuse? It's been explained to you over and over again. The resultant force on the water at the equator is still down as the centrifugal force (acting in the opposite directition to gravity) is tiny.

All the waters you see ARE curved. It's just that it's impossible to see as the radius of curvature is 6400 kilometres!

I'm beginning to think that this just one big wind up.

Did you look up Newton's laws?

None of you can show me the physical evidence form earth, can you? Impossible to see the curvature, can't find any bodies of water on earth dealing with centrifugal forces, the ground feels motionless.

"Did you look up Newton's laws?"

Did you look at the physical condition of earth?
We know the earth is round. Agree?
Title: Re: [Ask Me Anything] Will explain every observation using round earth model
Post by: Zaphod on April 06, 2017, 09:52:53 AM
"Did you look at the physical condition of the earth"

Yes, it's round. I can tell it's not flat because the sun sets below a clear horizon. (Not some wishy-washy fading away atmosphere bollocks). That horizon recedes if I gain altitude. The earth and sea do measurably bulge a bit at the equator due to centrifugal force.

Ok, did you look up Newton's laws?

Do you ever answer a question?
Title: Re: [Ask Me Anything] Will explain every observation using round earth model
Post by: physical observer on April 06, 2017, 10:05:19 AM
Physical Observer

Are you being deliberately obtuse? It's been explained to you over and over again. The resultant force on the water at the equator is still down as the centrifugal force (acting in the opposite directition to gravity) is tiny.

All the waters you see ARE curved. It's just that it's impossible to see as the radius of curvature is 6400 kilometres!

I'm beginning to think that this just one big wind up.

Did you look up Newton's laws?

None of you can show me the physical evidence form earth, can you? Impossible to see the curvature, can't find any bodies of water on earth dealing with centrifugal forces, the ground feels motionless.

"Did you look up Newton's laws?"

Did you look at the physical condition of earth?
We know the earth is round. Agree?

But not a spinning speeding sphere.
Title: Re: [Ask Me Anything] Will explain every observation using round earth model
Post by: physical observer on April 06, 2017, 10:06:15 AM
"Did you look at the physical condition of the earth"

Yes, it's round. I can tell it's not flat because the sun sets below a clear horizon. (Not some wishy-washy fading away atmosphere bollocks). That horizon recedes if I gain altitude. The earth and sea do measurably bulge a bit at the equator due to centrifugal force.

Ok, did you look up Newton's laws?

Do you ever answer a question?

"Yes, it's round."

But not a spinning speeding sphere.
Title: Re: [Ask Me Anything] Will explain every observation using round earth model
Post by: Zaphod on April 06, 2017, 10:12:40 AM
I give up.

Ah go on, one more go....

Did you look up Newton's Laws of motion? Have you tried to understand them - you must have done them at school. They explain all you misunderstandings and misconceptions.

Edit to add...

I'm doing forces/acceleration etc with my kids at the moment. I've been looking at that website I linked earlier. It really is excellent so thanks!

http://www.physicsclassroom.com/Physics-Tutorial/Newton-s-Laws
Title: Re: [Ask Me Anything] Will explain every observation using round earth model
Post by: physical observer on April 06, 2017, 10:31:58 AM
I give up.

Ah go on, one more go....

Did you look up Newton's Laws of motion? Have you tried to understand them - you must have done them at school. They explain all you misunderstandings and misconceptions.

Edit to add...

I'm doing forces/acceleration etc with my kids at the moment. I've been looking at that website I linked earlier. It really is excellent so thanks!

http://www.physicsclassroom.com/Physics-Tutorial/Newton-s-Laws

"I give up."

Smart move, because there is no visual evidence from earth's nature that supports being on a spinning speeding ball. I really don't care what Newton thought he experienced. Obviously, he didn't investigate the physical condition of earth that well.
Title: Re: [Ask Me Anything] Will explain every observation using round earth model
Post by: Zaphod on April 06, 2017, 10:44:48 AM
Ok, you're making yourself look ignorant and stupid.

The ignorance you can do something about with a bit of effort.

People are politely spoon feeding you answers but to not even look at the resources people are providing you is just plain rude.

Title: Re: [Ask Me Anything] Will explain every observation using round earth model
Post by: physical observer on April 06, 2017, 10:51:41 AM
Ok, you're making yourself look ignorant and stupid.

The ignorance you can do something about with a bit of effort.

People are politely spoon feeding you answers but to not even look at the resources people are providing you is just plain rude.

They are not the answers requested. It is you'll that is looking ignorant and stupid.
Title: Re: [Ask Me Anything] Will explain every observation using round earth model
Post by: Canadabear on April 06, 2017, 11:46:06 AM
Ok, you're making yourself look ignorant and stupid.

The ignorance you can do something about with a bit of effort.

People are politely spoon feeding you answers but to not even look at the resources people are providing you is just plain rude.

They are not the answers requested. It is you'll that is looking ignorant and stupid.

ignorant is somebody that ignores the evidence that got shown to him.
Title: Re: [Ask Me Anything] Will explain every observation using round earth model
Post by: Novarus on April 06, 2017, 11:51:26 AM
Ok, you're making yourself look ignorant and stupid.

The ignorance you can do something about with a bit of effort.

People are politely spoon feeding you answers but to not even look at the resources people are providing you is just plain rude.

They are not the answers requested. It is you'll that is looking ignorant and stupid.

ignorant is somebody that ignores the evidence that got shown to him.

Ignorant is also somebody who based their entire world view and makes assertions about the whole universe from a self-admitted incomplete perspective based on personal observation alone:

https://www.theflatearthsociety.org/forum/index.php?topic=69974.msg1893073#msg1893073
Title: Re: [Ask Me Anything] Will explain every observation using round earth model
Post by: inquisitive on April 06, 2017, 12:31:10 PM
Physical Observer

Are you being deliberately obtuse? It's been explained to you over and over again. The resultant force on the water at the equator is still down as the centrifugal force (acting in the opposite directition to gravity) is tiny.

All the waters you see ARE curved. It's just that it's impossible to see as the radius of curvature is 6400 kilometres!

I'm beginning to think that this just one big wind up.

Did you look up Newton's laws?

None of you can show me the physical evidence form earth, can you? Impossible to see the curvature, can't find any bodies of water on earth dealing with centrifugal forces, the ground feels motionless.

"Did you look up Newton's laws?"

Did you look at the physical condition of earth?
We know the earth is round. Agree?

But not a spinning speeding sphere.
Relative to the sun it is.
Title: Re: [Ask Me Anything] Will explain every observation using round earth model
Post by: rabinoz on April 06, 2017, 03:06:43 PM
Smart move, because there is no visual evidence from earth's nature that supports being on a spinning speeding ball. I really don't care what Newton thought he experienced. Obviously, he didn't investigate the physical condition of earth that well.

There is no logical reason why "visual evidence from earth's nature" should support it being on "a spinning speeding ball"!

There is plenty "visual evidence from earth's nature that supports being on a . . . . . . ball" and you been given plenty of that.

But, and you have been told this numerous times: Simple observation makes it clear that the earth rotating and/or moving relative to the moon, sun, planets and stars, and at different rates relative to all of them.

So, what had to be determined is:The present position is the latter, though the so-called "distant fixed stars" is more or less taken as a reference, even though there is no single point that is known to be stationary.

Being ignorant is quite understandable. Nobody can know everything, though some seem to think that they do.
But being wilfully ignorant is what can be called being stupid, though most of us are guilty of that in some areas.
Most of us tend to "close our eyes" to some things that we find are "inconvenient truths".

It is strange though that those who know the most are the ones that realise that there is so much more to find out:
(https://images.gr-assets.com/authors/1429114964p2/9810.jpg)....
“Two things are infinite: the universe and human stupidity; and I'm not sure about the universe.”
― Albert Einstein

So Mr Physical Observer, I choose not to follow you arbitrary rules because that would mean that I would be wilfully ignorant of information about the earth's motion.

You can choose to remain wilfully ignorant if you wish.

Title: Re: [Ask Me Anything] Will explain every observation using round earth model
Post by: frenat on April 06, 2017, 03:45:54 PM
It is you'll that is looking ignorant and stupid.
Says the guy STILL using "you'll" incorrectly after being told multiple times it does NOT mean "you all" but rather "you will".  Ah, irony.
Title: Re: [Ask Me Anything] Will explain every observation using round earth model
Post by: Mikey T. on April 06, 2017, 04:03:45 PM
It is you'll that is looking ignorant and stupid.
Says the guy STILL using "you'll" incorrectly after being told multiple times it does NOT mean "you all" but rather "you will".  Ah, irony.
Well, I can overlook grammatical errors if he could just get a basic grasp of critical thinking, at least at the level that my 7 year old autistic nephew has.
Title: Re: [Ask Me Anything] Will explain every observation using round earth model
Post by: JackBlack on April 06, 2017, 04:04:42 PM
That is not visual evidence that water on earth is being pulled to make earth's curvature, and also falling to a lower place on earth at the same time.
No one ever said anything of the like.
This is evidence that water can be acted upon by 2 forces at once, that gravity and centrifugal forces can both act to affect the water, going directly against your claim.

Water on earth does not act like the water in the rotating container. You can't find a body of water on earth that is both being pulled from earth by centrifugal force, and falling to a lower place on earth at the same time, can you? You can't even find a body of water on earth that is being pulled away from earth by centrifugal force, can you? That is why you need to insult and use lab experiments that do not apply.
We can, every single body of water on Earth is.
You can't find a single example where it isn't. The closest you can come are examples where one is not detectable.

This lab experiment disproves your claim and that both forces can act.

But no, both forces cannot dominate. You have 3 options:
Gravity wins and the water falls, but slower than if it wasn't spinning, and such that in equilibrium, it bulges at the equator.
Centrifugal forces win, and the water flies off the surface.
The 2 forces are balanced, and instead of falling or flying away it would orbit.


Notice the water in the rotating container becomes concave when centrifugal forces are applied, the water surface does not convex to make an arch, a bow across its surface, does it?
And that is because of the direction of those forces.
Gravity is pulling it down while the centrifugal forces pushes it to the side.
To represent Earth better you need a force pulling it towards the centre (which is what causes it to adopt a roughly spherical shape) and one pushing it out.

Is this example better:

It is water in micro-gravity, where it is held together by surface tension instead of gravity. The surface tension tries to make it a sphere, or to have the water "fall to the lowest point"
while

If you spun that container to match earth's 1,000 MPH surface
Then you would be spinning it way to fast. You cannot honestly represent the force by spinning a much smaller container at the same tangential velocity.
To honestly and accurately represent it you would need to spin it between 0.1 and 1 mile per hour.

I bet the water would fly out the open top of the container. You'd have to put a lid on it to keep the liquid in the container. Bodies of water on earth don't have lids on them, do they?
I see you are back to claiming it should fly off.

You really need to make up your mind.

Once I found wiki change the Principle of Biogenesis to the hypothesis of biogenesis
Before you were claiming it was a law. What changed? Did you realise you were wrong?

And yes, the wiki does try to correct itself, unlike FE.

Number two, show me an case from earth's nature where water is obeying both forces, centrifugal and gravity, at the same time, BET YOU CAN'T!!!!!!!!
We have provided you a case where water is obeying both forces. Care to address that?

"do you know how much centrifugal force is acting on the water?"

Enough to allegedly bow water on earth to make this:
No. Gravity is what does the majority of the bowing. Gravity alone would pull Earth into a sphere.
The centrifugal force results in a slight (0.15%) bulge. This acceleration is a mere 0.03 m/s^2 at the equator.


But water at the equator looks like this, horizontally flat to plane earth across its surface:
You keep saying that, but you are yet to prove it. You are yet to demonstrate that it doesn't follow the curve of Earth.

Ocean water Senegal, Africa, right on the equator. I don't see centrifugal forces at work:
Because they are too small for you to detect, and they are at an equilibrium with gravity.

I have shown you'll bodies of water at the equator, where the centrifugal forces are said to be the strongest, that do not match water being subjected to centrifugal forces. Water at the equator is flat, level and horizontal across its surface, it does not bow in a curve, and still fall to a lower spot on earth.
No. They do match. You don't seem to understand what that means do you?
Matching doesn't mean it is that force alone that is acting.
The water at the equator acts just as you would expect it to on this spinning speeding Earth.

You are also yet to show it is flat rather than following the curve of Earth.

It doesn't fall to the lowest point on Earth. It is higher than at the poles, because of the centrifugal force.

See, that is what you would need to do to show it doesn't match. You would need to show that the equatorial radius is the same as the polar radius, and this has to be done to a level of uncertainty where you would expect to see the difference.
You are yet to do that.
Instead you take a picture of a tiny piece and say it looks flat.
Guess what? That doesn't contradict Earth being a spinning speeding ball at all.

Smart move, because there is no visual evidence from earth's nature that supports being on a spinning speeding ball. I really don't care what Newton thought he experienced. Obviously, he didn't investigate the physical condition of earth that well.
No. There is plenty. It has been provided to you and you just dismiss it or lie about it or ignore it.

What there is no evidence of is Earth being flat and motionless.
Yes, there are some observations which are consistent with both, but that isn't evidence for either.
You are the one that is ignoring the physical condition of Earth.
Title: Re: [Ask Me Anything] Will explain every observation using round earth model
Post by: Jonny B Smart on April 07, 2017, 09:17:01 AM
Water conforms to forces acting upon it:

Title: Re: [Ask Me Anything] Will explain every observation using round earth model
Post by: Jonny B Smart on April 07, 2017, 09:18:48 AM
Another mountain where sun casts shadow upward. This makes FE model impossible. How can he sun be lower than the mountain?

https://www.reddit.com/r/mildlyinteresting/comments/5ba92u/mt_hood_casting_a_shadow_on_the_clouds_this/
Title: Re: [Ask Me Anything] Will explain every observation using round earth model
Post by: physical observer on April 07, 2017, 12:55:43 PM
Water conforms to forces acting upon it:



Now all you have to do is prove that that is what is happening to earth physically with evidence from physical earth, good luck!
Title: Re: [Ask Me Anything] Will explain every observation using round earth model
Post by: physical observer on April 07, 2017, 12:59:56 PM
Another mountain where sun casts shadow upward. This makes FE model impossible. How can he sun be lower than the mountain?

https://www.reddit.com/r/mildlyinteresting/comments/5ba92u/mt_hood_casting_a_shadow_on_the_clouds_this/

Go back and look at my rebuttal to the previous mountain shadow claim. It proves a motionless platform for the mountain, not one moving at 1,600 feet per second. The mountain would not be in the same position long enough to cast any shadow of any recordable duration on a spinning ball.

Mountain shadows proving rotating earth, debunked, again! You'll need a new shtick!
Title: Re: [Ask Me Anything] Will explain every observation using round earth model
Post by: frenat on April 07, 2017, 01:17:41 PM
Another mountain where sun casts shadow upward. This makes FE model impossible. How can he sun be lower than the mountain?

https://www.reddit.com/r/mildlyinteresting/comments/5ba92u/mt_hood_casting_a_shadow_on_the_clouds_this/

Go back and look at my rebuttal to the previous mountain shadow claim. It proves a motionless platform for the mountain, not one moving at 1,600 feet per second. The mountain would not be in the same position long enough to cast any shadow of any recordable duration on a spinning ball.

Mountain shadows proving rotating earth, debunked, again! You'll need a new shtick!
Because 1.5 degrees over 6 minutes is too fast?  Oh wait, no it isn't.  Just further proof you have no clue what you're talking about.
Title: Re: [Ask Me Anything] Will explain every observation using round earth model
Post by: JackBlack on April 07, 2017, 03:09:48 PM
Now all you have to do is prove that that is what is happening to earth physically with evidence from physical earth, good luck!
I already provided a picture of it.

Go back and look at my rebuttal to the previous mountain shadow claim. It proves a motionless platform for the mountain, not one moving at 1,600 feet per second. The mountain would not be in the same position long enough to cast any shadow of any recordable duration on a spinning ball.

Mountain shadows proving rotating earth, debunked, again! You'll need a new shtick!
Do you mean where you spoued pure nonsense and got your ass handed to you, yet again?

If so, the conclusion was the same as it will be here, the upcast shadow is evidence of a round Earth.

It does not prove that the mountain isn't moving.
Remember, the camera and clouds are moving as well.

You are yet to explain why it couldn't case the shadow and than just asserting nonsense.

How about you go and try to do the math for the shadow to see what it would be like on a spinning ball vs a motionless plane?
Or do you need someone else to do it for you?
Title: Re: [Ask Me Anything] Will explain every observation using round earth model
Post by: Yashas on April 09, 2017, 12:10:23 AM
I give up.

Ah go on, one more go....

Did you look up Newton's Laws of motion? Have you tried to understand them - you must have done them at school. They explain all you misunderstandings and misconceptions.

Edit to add...

I'm doing forces/acceleration etc with my kids at the moment. I've been looking at that website I linked earlier. It really is excellent so thanks!

http://www.physicsclassroom.com/Physics-Tutorial/Newton-s-Laws

"I give up."

Smart move, because there is no visual evidence from earth's nature that supports being on a spinning speeding ball. I really don't care what Newton thought he experienced. Obviously, he didn't investigate the physical condition of earth that well.

http://physics.stackexchange.com/questions/209805/rivers-that-flow-uphill-due-to-earths-rotation

Oopps. Some rivers flow upwards because the centrifugal force isn't exactly opposite to the gravitational force.

RIP.
Title: Re: [Ask Me Anything] Will explain every observation using round earth model
Post by: disputeone on April 09, 2017, 12:14:52 AM
That's funny curvature shouldn't be measurable left to right at sea level.

I've seen people measure a horizon drop with a theodolite but I think you don't know what you are talking about.

Whats next? Toilets flushing in different directions? Travelling west in a plane is faster than travelling east?

Edit, in hindsight it looks like the picture is on the piss, check your line again.

Travelling west in a plane is faster than travelling east because the earth is rotating. Go find out why rockets are launched from west to east. *high school physics*

Lmao.

Dude weak.

Inertial FoR, high school physics.

Can you elaborate on your claim of magic planes?

(What is it with people and magic planes?)
Title: Re: [Ask Me Anything] Will explain every observation using round earth model
Post by: Yashas on April 09, 2017, 12:40:21 AM
That's funny curvature shouldn't be measurable left to right at sea level.

I've seen people measure a horizon drop with a theodolite but I think you don't know what you are talking about.

Whats next? Toilets flushing in different directions? Travelling west in a plane is faster than travelling east?

Edit, in hindsight it looks like the picture is on the piss, check your line again.

Travelling west in a plane is faster than travelling east because the earth is rotating. Go find out why rockets are launched from west to east. *high school physics*

Lmao.

Dude weak.

Inertial FoR, high school physics.

Can you elaborate on your claim of magic planes?

(What is it with people and magic planes?)

The direction of jetstreams are decided by the rotation of the earth. I am sorry for not being clear. If there was no jetstream, travelling either side in a plane wouldn't make a difference.
Title: Re: [Ask Me Anything] Will explain every observation using round earth model
Post by: rabinoz on April 09, 2017, 01:01:47 AM
Another mountain where sun casts shadow upward. This makes FE model impossible. How can he sun be lower than the mountain?

https://www.reddit.com/r/mildlyinteresting/comments/5ba92u/mt_hood_casting_a_shadow_on_the_clouds_this/

Go back and look at my rebuttal to the previous mountain shadow claim. It proves a motionless platform for the mountain, not one moving at 1,600 feet per second. The mountain would not be in the same position long enough to cast any shadow of any recordable duration on a spinning ball.

Mountain shadows proving rotating earth, debunked, again! You'll need a new shtick!
You did not debunk anything! You simply ignore any and all answers showing that your claims are bunkum.
Title: Re: [Ask Me Anything] Will explain every observation using round earth model
Post by: Copper Knickers on April 09, 2017, 01:45:16 PM
That's funny curvature shouldn't be measurable left to right at sea level.

I've seen people measure a horizon drop with a theodolite but I think you don't know what you are talking about.

Whats next? Toilets flushing in different directions? Travelling west in a plane is faster than travelling east?

Edit, in hindsight it looks like the picture is on the piss, check your line again.

Travelling west in a plane is faster than travelling east because the earth is rotating. Go find out why rockets are launched from west to east. *high school physics*

Lmao.

Dude weak.

Inertial FoR, high school physics.

Can you elaborate on your claim of magic planes?

(What is it with people and magic planes?)

The direction of jetstreams are decided by the rotation of the earth. I am sorry for not being clear. If there was no jetstream, travelling either side in a plane wouldn't make a difference.

Indeed, and jet streams go from west to east. So travelling west in a plane is very often slower than travelling east (as I mentioned (https://www.theflatearthsociety.org/forum/index.php?topic=70085.msg1894097#msg1894097) in the other thread).